PLEASE HELP ME THIS IS URGENT I AM BEING TIMED!!!

A book has 280 pages. Of the 280 pages, 210 have printing, 40 have pictures, and 30 are blank. Write your answers as fractions. Represent a fraction using the / symbol.

What is the probability of not opening to a printed page?

Answers

Answer 1

Answer:

1/4

Step-by-step explanation:

printed part = 210

total pages = 280

probability to open printed page 210/280 =3/4

probability to not open printed page = 1 - probability to open printed page = 1 - 3/4 = 1/4

Answer 2

Answer:

1/4

Step-by-step explanation:

280-210=70

P=70/280=1/4


Related Questions

Alison, the owner of a regional chain of pizza stores, is trying to decide whether to add calzones to the menu. She conducts a survey of 700 people in the region and asks whether they would order calzones if they were on the menu. 46 people responded “yes.” Create a 90% confidence interval for the proportion of people in the region who would order calzones if they were on the menu.Round your answer to four decimal places.

Answers

Answer:

Step-by-step explanation:

Confidence interval is written as

Sample proportion ± margin of error

Margin of error = z × √pq/n

Where

z represents the z score corresponding to the confidence level

p = sample proportion. It also means probability of success

q = probability of failure

q = 1 - p

p = x/n

Where

n represents the number of samples

x represents the number of success

From the information given,

n = 700

x = 46

p = 46/700 = 0.066

q = 1 - 0.066 = 0.934

To determine the z score, we subtract the confidence level from 100% to get α

α = 1 - 0.9 = 0.1

α/2 = 0.1/2 = 0.05

This is the area in each tail. Since we want the area in the middle, it becomes

1 - 0.05 = 0.95

The z score corresponding to the area on the z table is 1.645. Thus, the z score for a confidence level of 90% is 1.645

Therefore, the 90% confidence interval is

0.066 ± 1.645√(0.066)(0.934)/700

= 0.066 ± 0.0094

The lower limit of the confidence interval is

0.066 - 0.0094 = 0.0566

The upper limit of the confidence interval is

0.066 + 0.0094 = 0.0754

Answer:

(0.0503, 0.0811).

Step-by-step explanation:

The confidence interval for the unknown population proportion p is (p^−z⋆p^(1−p^)n−−−−−−−−√,p^−z⋆p^(1−p^)n−−−−−−−−√). The confidence interval can be calculated using Excel.

1. Identify α. Click on cell A1 and enter =1−0.90 and press ENTER.

2. Thus, α=0.1. Enter the number of successes, x=46, and sample size, n=700, in the Excel sheet in cells A2 and A3. To find the proportion of successes, p^, click on cell A4 and enter =A2/A3 and press ENTER.

3. Thus, p^≈0.0657. Use the NORM.S.INV function in Excel to find z⋆. Click on cell A5, enter =NORM.S.INV(1−A1/2), and press ENTER.

4. The answer for z⋆, rounded to two decimal places, is z⋆≈1.64. To calculate the standard error, p^(1−p^)n−−−−−−−−√, click on cell A6 and enter =SQRT(A4∗(1−A4)/A3) and press ENTER.

5. The answer for the standard error, rounded to four decimal places, is p^(1−p^)n−−−−−−−−√≈0.0094. To calculate the margin of error, z⋆p^(1−p^)n−−−−−−−−√, click on cell A7 and enter =A5*A6 and press ENTER.

6. The answer for the margin of error, rounded to four decimal places, is z⋆p^(1−p^)n−−−−−−−−√≈0.0154. The confidence interval for the population proportion has a lower limit of A4−A7=0.0503 and an upper limit of A4+A7=0.0811. Thus, the 90% confidence interval for the population proportion of people in the region who would order calzones if they were on the menu, based on this sample, is approximately (0.0503, 0.0811).

Calculate the radius of the circular rug. Use 3.14 for π.







Enter your answer in the box below.

____ feet









Answers

Answer:

[tex]\sqrt{\frac{153.86}{3.14} } = r[/tex]

Step-by-step explanation:

a=pir^2

153.86 = 3.14*r^2

[tex]\sqrt{\frac{153.86}{3.14} } = r[/tex]

You're welcome and don't forget to click THANKS

Answer:

\sqrt{\frac{153.86}{3.14} } = r

Step-by-step explanation:

a=pir^2

153.86 = 3.14*r^2

\sqrt{\frac{153.86}{3.14} } = r

Find an equation of the line passing through the pair of points. Write the equation in the form Ax+By = C.
(-6,3), (-7, - 7)

Answers

Answer:

-10x + y = 63

Step-by-step explanation:

Let's find the slope of this line.  Going from (-7, -7) to (-6, 3), the "run" is +1 (this is the increase in x) and the "rise" is -10 (y increases by 10).  Thus, the slope, m, is m = rise / run = 10 / 1 = 10.

Using the point-slope version of the equation of a straight line, we get

y + 7 = 10(x + 7)

Performing the multiplication results in

y + 7 = 10x  + 70.

Rearranging these terms in the form Ax + By = C, we get:

-10x + y = 63

Check this by substituting -6 for x and 3 for y:

-10(-6) + 3 = 63     This is TRUE

Help pleasseee •-•
1. The dot plot below shows the vertical jump of some NBA players. A vertical jump
is how high a player can jump from a standstill. Draw a box plot of the heights for the
vertical jumps of the NBA players above the dot plot.

Answers

The picture below shows the box plot.

Which of the following expressions can be factored by grouping? (Select all that apply.)

a^2 - 9 a + 7 ab + 63 b
3 a + 4 ab - b - 12
ab + 6 b - 2 a - 12
x^3 + 9 x^2 + 7 x + 63
ay^2 + a - y^2 - 1

Answers

Answer:

Options (3), (4) and (5)

Step-by-step explanation:

1). a² - 9a + 7ab + 63b

  = a(a - 9) + 7b(a + 9)

  Now we can not solve this problem further.

  Therefore, can't be factored by grouping.

2). 3a + 4ab - b - 12

   = a(3 + 4b) - 1(b - 12)

   We can't solve it further.

   Therefore, can't be factored by grouping.

3). ab + 6b - 2a - 12

   = b(a + 6) - 2(a + 6)

   = (b - 2)(a + 6)

  We can be factored this expression by grouping.

4). x³ + 9x²+ 7x + 63

   = x²(x + 9) + 7(x + 9)

   = (x² + 7)(x + 9)

   Therefore, the given expression can be factored by grouping.

5). ay² + a - y² - 1

  = a(y² + 1) - 1(y² + 1)

  = (a - 1)(y² + 1)

  This expression can be factored by the grouping method.

Options (3), (4) and (5) are the correct answers.

Ellen thinks that if a line has no slope, then it never touches the y-axis. Which line proves that her statement is incorrect?
A. x = 0
B. y = 0
C. x = 1
D. y = 1

Answers

Answer:

Option D

Step-by-step explanation:

A horizontal line has no slope. The equation 'y=1' graphs as a horizontal line. It passes (0,1). So, it touches the y-axis.

A line with an undefined slope does not touch the y-axis. It is vertical.

Option D should be the correct answer.

For the graph, what is a reasonable constraint so that the function is at least 300?
(Graph Below)

A) x ≥ 0

B) −5 ≤ x ≤ 30

C) 0 ≤ x ≤ 25

D) All real numbers

Answers

Answer:

C) 0 ≤ x ≤ 25

Step-by-step explanation:

We are supposed to find a reasonable constraint so that the function is at least 300 i.e. the value of x at which f(x) is greater or equal to 300

A)x ≥ 0

Refer the graph

At x = 0

f(x)=300

On increasing the value of x , f(x) increases but at x = 12 it starts decreasing

So, x ≥ 0 can also have f(x)<300

So, Option A is wrong

B)−5 ≤ x ≤ 30

At x = -5

f(x) = 100

So, Option B is wrong since we require f(x) is greater or equal to 300

c)0 ≤ x ≤ 25

At x = 0

f(x)=300

At x = 12 , it starts decreasing

At x = 25

f(x)=300

So, The value of f(x) is at least 300 when 0 ≤ x ≤ 25

D)All real numbers

At x = 30

f(x)=0

But we require f(x) greater or equal to 300

Hence Option C is true

Answer:

C) 0 ≤ x ≤ 25

Its correct! Hope this helps :)

In the diagram, measure angle 3 = 120° and measure angle 12 = 80°. Which
angle measures are correct? Check all that apply
measure angle 1 = 60°
measure angle 13 = 80
measure angle 6 = 80°
measure angle 5 = 60°
measure angle 10 = 120
measure angle 14 = 100

Answers

Answer:

1, 13, 5 are correct

Step-by-step explanation:

Answer: A,B,D,F are correct

Step-by-step explanation:

Mrs. Burgess, Mrs. Curry, Ms. Davis and Mrs.
Stowers went to lunch this weekend. The lunch
total was $81.20. How much does each person
owe if they split it evenly?

Answers

Answer:

$20.30

Step-by-step explanation:

There are 4 people so 81.20/4 = 20.30

Answer: $20.30

Step-by-step explanation:

There are 4 people and so you need to divide the cost by the amount of people to see what each person owes

Bob Farted 3 times every 30 seconds how many time does Bob fart in 2 days?

Answers

The answer would be 17280

Answer:

he would fart 17280 times in 2 days

Step-by-step explanation:

Solve for the missing value x 3:9=x:81

Answers

Answer:

x=27

Step-by-step explanation:

So let's look at the ratio

3:9

x:81

x is proportional to 3 the same way that 81 is to 9

How would we get 9 to 81?

We would multiply by 9

So what is 3*9?

27

The shorter leg of a right triangle is 21 feet less than the other leg. Find the length of the two legs if the hypotenuse is 39 feet.

Answers

Answer:

leg 1= 36  feet

leg2 = 15  feet

Step-by-step explanation:

Hi, we have to apply the Pythagorean Theorem:  

c^2 = a^2 + b^2  

Where c is the hypotenuse of the triangle (in this case the distance between Doreen’s house and the tower) and a and b are the other legs.  

leg1 = x

leg2 = x-21 (21 feet less than the other leg)

Replacing with the values given:  

39^2 = x^2 + (x-21)^2  

1,521 = x^2 + x^2 -42x +441

0 = 2x^2 -42x +441-1,521

0= 2x^2 -42x -1,080

For: ax2+ bx + c  

x =[ -b ± √b²-4ac] /2a  (quadratic formula)

Replacing with the values given:  

x=-(-42)± √(-42)²-4(2)-1080] /2(2)  

x= 42± √10,404] /4

x = 42± 102 /4

Positive:

x = 42+102 /4 = 36

leg 1= 36

leg2 = 36-21 =15

Feel free to ask for more if needed or if you did not understand something.  

The length of shorter leg is 15 feet and other leg is 36 feet.

Pythagoras theorem:

Let us consider that, length of shorter leg is x feet.

So that, length of other leg is [tex](x+21)[/tex] feet.

It is given that, the hypotenuse is 39 feet.

Apply Pythagoras theorem,

                 [tex]x^{2} +(x+21)^{2} =(39)^{2}\\ \\x^{2} +x^{2} +441+42x=1521\\\\2x^{2} +42x-1080=0\\\\x=15 ,-36[/tex]

Since, side can not be negative.

So that, we have to consider x=15 feet.

Length of other leg[tex]=x+21=15+21=36feet[/tex]

Learn more about the Pythagoras theorem here:

https://brainly.com/question/343682

WILL GIVE BRAINLIEST AND 30 POINTS! Simplify −3 +i over 7 + i . First, multiply both the numerator and denominator by A. -3 + i B. -3 -i C. 7 + i D. 7-1 Hint: C is incorrect.

Answers

You have (-3 +i)/(7 + i) so you need to remove the i from the numerator, so you have to multiply by 7-i and you get

(-22 + 4i)/50 which can be simplified to

(-11 + 2i)/25

Answer:

Step-by-step explanation:

What you are trying to do in this question is get rid of the i in the denominator. To do that you must multiply top and bottom by the conjugate of 7 + i

The conjugate is 7 - i

D if correct if you just made a typo. Otherwise there is no answer.

( - 3 + i ) * (7 - i)

=============

(7 + i ) ( 7 - i)

( - 3 + i ) * (7 - i)

=============

49 - i^2

( - 3 + i ) * (7 - i)

=============

49 + 1

( - 3 + i ) * (7 - i)

=============

50

I'll let you work out the numerator. If you get (-2 + i)/5 you are correct.

What is the third quartile of this data set 14,18,20,21,25,32,42,46,48

Answers

Answer:

40

Step-by-step explanation:

Divide the list of numbers in half.

Since there are 9 numbers (odd number), divide like this :

14, 18, 20, 21      25       32, 42, 46, 48

Find the median of the third one. 

(46 + 42)/2 = 44

So the answer is 44

Hope this helped!

please mark me brainliest :)


Thomasina wrote down a number that is both rational and a whole number. What is one possible number she could have written down?

Answers

Answer:

1

Step-by-step explanation:

Answer:

2.

Step-by-step explanation:

Rational numbers can be written as a/b where a and b are integers ( b not zero) and a and b are co-prime ( no common factors)

A whole number is rational (2 for example can be written as 2/1).

It could have been 2 or 3 or 33.

Use the discriminant to determine the number of real solutions to the quadratic equation. 11m2+12m+4=0 What is the number of real solutions? Select the correct answer below: 0 1 2

Answers

Answer:

0

Step-by-step explanation:

The discriminant is

12^2 - 4*11*4 = -32

Since it is negative, there are 0 real solutions

What is the solution for x in the equation? 4x − 3 + 5 = 2x + 7 − 8x

Answers

Answer:

4x+2= -6x+7

4x+6x=7-2

10x=5

x=1/2

Step-by-step explanation:

The answer is x=.5 you can download and use Photomath to scan the question and that will tell you how to do its step by step

Write the set of real numbers x less than 6 in set builder notation

Answers

Answer:

{x ∈ R: x<6}

Step-by-step explanation:

Given

x is a real numberx is less than 6

Required

Write the set using set builder notation

The very first thing to do is to list out the range of x, using inequalities;

x is less than 6 implies that -infiniti < x < 6

The next step is to translate this to set builder. This is done as follows

x ∈ R - > This means that x is a real number

x < 6 -> where x is less than 6.

Bringing these two together, it gives:

{x ∈ R: x<6}

Hence, the set of real numbers x less than 6 is equivalent to {x ∈ R: x<6} using set builder notation

Stock Texas has a price of $156 per share when Bond Austin has a price of $23 per bond. Use an
equation modeling the inverse variation between the stock and bond prices to predict the price of
Stock Texas when Bond Austin is worth the following
$68.74​

Answers

Answer:

I think the answer would be $466.24.

What is the domain of the function y = RootIndex 3 StartRoot x EndRoot?
Negative infinity less-than x less-than infinity
0 less-than x less-than infinity
0 less-than-or-equal-to x less-than infinity
1 less-than-or-equal-to x less-than infinity

Answers

Answer:

A

All real number which would be -infinity < x < infinity

The domain of the function is the set of all real numbers given by; Negative infinity less-than x less-than infinity.

What is the domain of the function?

According to the task content;

It follows that the function whose domain is to be defined is; y = ³√x.

Since, the domain of a function includes all possible values of its input variables (x in this case). It follows that the domain of the function in discuss is; all real number values. This follows from the fact that all the cube root of all real numbers can be evaluated.

Read more on domain;

https://brainly.com/question/2264373

#SPJ2

ASAP Mario was given the following enlargement. Side A corresponds to a side with length 24 centimeters. A side with length 6 centimeters corresponds with a side with length 12 centimeters. Figures not drawn to scale. What is the length of side A, in centimeters? 12 15 24 33

Answers

Answer:

The length of side A is 12 cm

Step-by-step explanation:

This question is best answered when there's an attachment; however, the absence of an attachment doesn't mean it can't be solved.

Given

Side A corresponds to 24 cm

Length 6 cm corresponds to Length 12 cm

Required

Find A

The given parameters can be represented mathematically as;

[tex]\frac{A}{24} = \frac{6}{12}[/tex]

With this expression, the value of A can be easily solved

First; multiply both sides by 24

[tex]24 * \frac{A}{24} = \frac{6}{12} * 24[/tex]

[tex]\frac{24 *A}{24} = \frac{6}{12} * 24[/tex]

[tex]\frac{24A}{24} = \frac{6}{12} * 24[/tex]

[tex]A = \frac{6}{12} * 24[/tex]

[tex]A = \frac{6 * 24}{12}[/tex]

[tex]A = \frac{144}{12}[/tex]

[tex]A =12[/tex]

Hence, the length of side A is 12 cm

Answer:

ANSWER:A

Step-by-step explanation:

ANSWER IS A!SAVE YOURSELF! RUN! DON'T FAIL UR TEST!

byeeeeee:)

<3

HELP ME PLZ!!!
In Tier 3, half of the shelf space will occupy Go Math Textbooks and the other half of the space with Bounty paper towels, as shown below. Your answer must have work on how you did it OR an explanation.

Answers

Answer:

Your in middle school you should know this figure this by now FIGURE IT OUT ON YOUR OWN.....

Step-by-step explanation: yeah

Answer:

i dont know the volume but if it tells you what each book measures. Ive left a guide below to distribution.

Step-by-step explanation:

volume = 1

0.5 +0.5 = 1

volume 2

1+1 = 2

you just need to use the word distribution and show like this

(V): amount  / 2 =  new amount (adding cm^3)

To work out volume you take the length and width and height of the shelf of the books. you show your workings like this.

(V): L x Wx H =

You then multiply the amount of books if you was not given the measure of the bookcase shelf.

IF the bookcase is double like the picture and doesnt show the shelf measure than you half the hieght only and keep the same length and width.

H/2 =   first and enter below the new height.

(V): L xWx H  = this is the whole shelf

(V) =     / 2   you enter the Volume amount infront of the /2 and this way you see the volume for the math textbooks being 1/2 of 1  by (/2)  

Use the found answer

LAbel

MAth text books (V) =

Bounty paper towells (V) =

Then see how many cubic sheets fit.

ie) if they measure 30cm x 1cm x 30xm

and the shelf measures 30cm x 60 x 60

you can count up from the second values as the first values fit the length

The width = 1 x 60 - 60 books.  (example 1cm Width thickness)

Then see the height does not change to the height of the books.

IF it does change then for every 2cm change you divide the height of the bookcase by 60-1, 60-2 or 60-3, 60-4cm etc. just one of these that fits the indifference of height

then multiply if 60-58cm = 58 we know this is 2 / 60 x 100 = 3.33%

To calculate percentage decrease: see above

First: work out the difference (decrease) between the two numbers you are comparing. Then: divide the decrease by the original number and multiply the answer by 100. If your answer is a negative number, then this is a percentage increase.

You then apply (V) of shelf number x 0.03 and then subtract from that shelf volume to show your workings or reapply the same technique to see how many fit as decrease ie) 0.50% =  amount x 0.50 = 1/2 new amount.

WHY CAN'T ANYONE HELP ME? 1. In Michigan, the sales tax rate is 6%. Suppose that an item costs $50.88 including tax. How can we use algebra to find the price of the item before tax was added? What is the price before tax? Note: Using x = the price before tax, write an equation, and show solving steps to find x. (SOLVE AND CHECK)

Answers

Answer: $47.83

Step-by-step explanation: Let's turn the percentage into a decimal, move the decimal two times to the right...

6% ⇒ 0.06

0.06 * $50.88 = $3.05

$50.88 - $3.05 = $47.83

Remember x = price before tax...

x = $47.83

10. A cliff on the seashore is eroding at the rate of 17 centimeters per year. Write and solve an equation to find the number of
years in which the cliff will erode 85 centimeters.

Answers

Answer:

The number of  years in which the cliff will erode 85 centimeters are:

5 years.

Step-by-step explanation:

To solve the question in the text, you must create a formula that includes the rate (17 centimeters per year) and a variable representing the time (t), how I show below:

f(t) = 17t

Where:

t = time in years

Remember that where are not symbols between two numbers o variables, it means you must multiply (17t actually is 17 * t). Now you only must replace t by years 1 per 1 until obtain the value of 85 (the eroding wanted):

f(1) = 17(1) = 17 f(2) = 17(2) = 34 f(3) = 17(3) = 51 f(4) = 17(4) = 68 f(5) = 17(5) = 85

How you can see, when the time is equal to 5, the f(t) or the eroding is 85, the value we wanted.

help me, please.........

Answers

Answer:

de for the second

Step-by-step explanation:

Answer:

Step-by-step explanation:

an  altitude   -  a ( forms with a line an angle of 90 degrees)

a perpendicular bisector-  c ( divide the line in 2 equal parts and it is perpendicular on  the line)

an angle bisector  - d ( divide the angle in 2 equal parts)

a median - b ( a segment between an angle and the middlepoint of the opposite line)

4 + 5 /3 squared - 2 squared

Answers

Answer:

[tex]\frac{25}{9}[/tex]

Step-by-step explanation:

[tex]4+\frac{5}{3} ^2-2^2=\frac{5}{3}^2=\frac{25}{9}[/tex]

The answer is 25/3 as the final answer

does anyone know the answer to this ?

Answers

Step-by-step explanation:

The circumference of the clock will be 8.6π because its hand is the radius. In 15 minutes the hand moves a quarter of the circumference which is 2.15π or 6.571.

Can someone help me on these two questions?

Answers

Answer:

1. 88 degrees

2. x=8

Step-by-step explanation:

1.

In a hexagon, all of the interior angles must add up to 720 degrees. Therefore:

x+125+120+95+120+172=720

x=88

2.

Since opposite sides of a parallelogram are equal:

2x=x+8

x=3x-16

Let's go with the first equation.

2x=x+8

Subtract x from both sides:

x=8

You can plug this into the second equation to ensure it works:

3(8)-16=8

24-16=8

8=8

Hope this helps!

Answer:

1. 88

2. 48

Step-by-step explanation:

1. The total of the inner angles of the hexagon is 720

95+120+120+172+125=632

720-632=88

2. 2x=x+8

x=8

If we put all the x's in place and add them, the result is: 48

Find the cardinal number for the set. Α = {3, 6, 9, . . . , 36}

Answers

Hey mate!!

Answer:

n(A)=12

Solution,

A={3,6,9,12,15,18,21,24,27,30,33,36}

Just count the elements,

There are 12 elements in the set.

So the cardinal number of the set is 12.

Hope it helps...

Answer:

[tex]12[/tex]

Step-by-step explanation:

A = {3,6,9,12,15,18,21,24,27,30,33,36}

To find the cardinal number,

just count the numbers

So, according to that,

there are 12 numbers in this set.

So, it's clear for us that, the cardinal number of this set is 12

n (A)= 12

hope this helps

brainliest appreciated

good luck! have a nice day!

Evaluate 3x for x = −2, x = 1, and x = 3. Question 18 options: A) 1∕3, 0, 9 B) 1∕9, 3, 27 C) 9, 3, 27 D) 1∕9, 9, 27

Answers

Answer:

D

Step-by-step explanation:

Took it on Ed but it might be different

Answer:

D

Step-by-step explanation:

I did the math :)

Other Questions
Match the correct President to the event:1. Ford2. Carter3. Reagan4. George H. W. Bush5. Clinton6. George W. Bush7. ObamaA) passage of 780-billion dollar stimulus packageB) Roe vs. WadeC) Inflation skyrocketedD) end to the cold warE) the defense of marriage ActF) known as the Great communicatorG) Attacks of 9/11 Maya wrote a program and forgot to put the steps in the correct order. Which step does Maya need to review? Pattern following Planning Segmenting Sequencing Which of the following is(are) the solution(s) to | x-1|-8?A. X= 7.-9B. X = 9C. X = -79D. X = 7 which of the following explains expressions are equivalent to - 5/6 /-1/3 Listen to those tunes you will be amazed.A. tunes, youB. tunes and youC. correct as isD. If you listen to those tunes, you If cos 67 degrees is close to 2/5 which is closest to the length of noPO=100cm P=23 degrees Ethan is concerned because his boyfriend has tested positive for an STI. His friend Kaitlyn recommends that he visit his doctor or urgent care clinic for testing. Another friend, Jamal, suggests a local health clinic he found on the Internet. Jamal explains that the price for testing and medication could be expensive, but there are programs available to help reduce the cost. Ethan decides to try out the local clinic. The doctor there is very encouraging but explains that Ethan has also contracted chlamydia. The doctor explains how to use condoms to prevent the spread of STIs in the future, and writes Ethan a prescription for antibiotics. He also recommends a follow-up visit for retesting. Which statement summarizes Ethans experience? It is difficult to be proactive about treatment because limited resources are available. The cost associated with testing and treating an STI is minimal. Social stigma can be experienced from family, friends, and healthcare providers. Affordable resources are available to treat sexually transmitted infections. Based on the pedigree that is shown above, how to describe Fred? Which expression is represented by the number line?A number line going from negative 4 to positive 4. An arrow goes from negative 2.5 to negative 1, from 0 to 3, and from 3 to negative 2.5.3 5.5 1.53 5.5 + 1.53 2.5 13 2.5 + 1 What is the Arrhenius definition of an acid? A substance that increases H3O+ concentration when it is dissolved in water. A substance that increases OH concentration when it is dissolved in water. A compound that donates protons. A compound that accepts protons. Nucleic acids and carbohydrates are both types of what?A. MacromoleculesB. ProteinsC. ElementsD. AtomsSUBM cuanto es r2-2r-7=0 If the following words were arranged to make the best sentence, with what letter would the fifth word begin? YARDS IS A TEN HUNDRED TO POSSIBLE SECONDS IN RUN ITrasthThe correct answer was "It is possible to run ten yards in a hundred seconds . The correct answer is "r" Two students fertilize a pair of pea plants. One plant is Rr, and the other is rr. The offspring produced bythese two plants all had the dominant trait. The Punnett square showed that there was a 50 percentchance of the offspring carrying rr genes. So, the student assumes that he made an error in determiningW the genetic makeup of the plants. Do you think the student made an error, or is there anotherexplanation? i need help, please.. Noah visits different states every summer. This summer, he started with 2 states already visited and visits 6 states per day. Write an equation to model this situation (use d for days and s for states) 2 PontsThe estimate obtained from a sample of which of the following sizes wouldmost likely be closest to the actual parameter value of a population?A. 15B. 150C. 75D. 45SUBM Which country is most likely to offer women more rights?AfghanistanIranNorwaySaudi Arabia Hey so im really confused on how to rewrite problems into radical form, so I was wondering if anyone would be able to show me how to do It step by step? 1. Which statement correctly completes this timeline? *East-West Conflict in Europe During the Cold War1946-1949The Soviet Union forms the communist Eastern Bloc.1947The United States announces the Truman Plan.1948-1949The Soviet Union enforces the Berlin Blockade.1948-19491949Western Powers form the North Atlantic Treaty Organi